Go Back   StudyChaCha 2024 2025 > StudyChaCha Discussion Forum > General Topics

  #1  
Old April 30th, 2014, 05:20 PM
Unregistered
Guest
 
Default Graduate Management Admission Test previous year question papers

Can you please give me the Graduate Management Admission Test previous year question papers as it is very urgent for me?
Reply With Quote
Other Discussions related to this topic
Thread
Common Law Admission Test previous year question papers
UGC National Eligibility Test Management previous year question papers
Management Aptitude Test previous year question papers in PDF format free download
Air Force Common Admission Test previous year question papers
Graduate Aptitude Test in Engineering previous year question papers of all the stream
Common Admission Test previous year question papers free download in PDF format
Graduate Aptitude Test in Engineering Physics previous year question papers free down
UGC National Eligibility Test previous year question papers for management
Graduate Management Admission Test previous year question papers free download
Air Force Common Admission Test 2014 previous year question papers
Common Management Aptitude Test previous year question papers
Graduate Aptitude Test in Engineering previous year question papers of ECE free downl
Graduate Pharmacy Aptitude Test previous year question papers free download
Graduate Aptitude Test in Engineering previous year question papers free download in
Management Aptitude Test previous year question papers in PDF format
IIT Joint Admission Test previous year question papers free download
Common Admission Test previous year question papers
Common Law Admission Test previous year question papers for UG courses
Graduate Aptitude Test in Engineering previous 10 year question papers free download
Common Management Admission Test previous year question papers






  #2  
Old May 1st, 2014, 04:48 PM
Sashwat's Avatar
Super Moderator
 
Join Date: Jun 2011
Default Re: Graduate Management Admission Test previous year question papers

As you want to get the Graduate Management Admission Test previous year question papers so here is the information of the same for you:

Section 1 - Verbal Ability
No.of Questions : 20
Duration in Minutes: 20
Assessments by Merit Trac
Directions for Questions 1-3 : Choose the option which will correctly fill the blank.
1) I am writing to enquire _________the possibility of hiring a conference room at the hotel on the 2nd
of September.
A) Of
B) About
C) Into
D) After
2) _________ having her lunch, she stood - the tree and waited _______ him.
A) With, below, for
B) After, under, for
C) Inside, further, to
D) About, across, into
3) The microscopic animals are the primary food for larval cod and their decline has meant that fewer
fish are making it to adulthood to be caught_________ trawlermen.
A) In
B) Into
C) By
D) With
Directions for Questions 4-6 : Choose the word nearest in meaning to the word in ITALICS from the
given options.
4) The jacket is impervious to water.
A) Dirty
B) Pure
C) Impenetrable
D) Favorable
5) Chandan was chagrined with the continuous disruption of the power supply to his home.
A) Delighted
B) Creation
C) Peeved
D) Security
6) The latest ordinance issued by the government has provided the bank with two options.
A) Decision
B) Law
C) Opinion
D) Verdict
Directions for Questions 7-10: Choose the answer option which will correctly fill the blank.
7)_________ great writer is convinced that whatever he says is not an echo or imitation of what others
have said.
A) An
B) The
C)A
D) No article required
8) ________ Reserve Bank of India directed banks to closely watch _______spending through
International Debit Cards.
A) A,the
B) The, the
C) The, a
D) -\n, the
9) The officer received _____ official letter from _____ Ministry of IT in _____ Central Government.
A) A, the, an
C) An, the, the
B) A, an, the
D) An, an, the
10) You CANNOT send out ______uneducated man into ______ world of technology and expect him
to perform.
A) An, an
B) A, an
C ) An, the
D) The, an
Directions for Questions 11-15: Read the passage and answer the questions that follow on the basis of
the information provided in the passage. Microprocessor is an electronic computer Central Processing
Unit (CPU) made from miniaturized transistors and other circuit elements on a single semiconductor
Integrated Circuit (IC). Before the advent of microprocessors, electronic CPUs were made from
individual small-scale Integrated Circuits containing the equivalent of only a few transistors. By
integrating the processor onto one or a very few large-scale Integrated Circuit packages (containing the
equivalent of thousands or millions of discrete transistors), the cost of processor power was greatly
reduced. The evolution of microprocessors has been known to follow Moore's Law when it comes to
steadily increasing performance over the years.
This law suggests that the complexity of an Integrated Circuit with respect to minimum component cost
will double in about 18 months. From humble beginnings as the drivers for calculators, the continued
increase in power has led to the dominance of microprocessors over every other form of computer;
every system from the largest mainframes to the smallest handheld computers now uses a
microprocessor at their core. .As with many advances in technology, the microprocessor was an idea
wbose time had come. Three projects arguably delivered a complete microprocessor at about the same
time: Intel's 4004, Texas Instruments' TMS1000, and Garrett AiResearch's Central Air Data Computer. .
A computer-on-a-chip is a variation of a microprocessor, which combines the microprocessor core
(CPU), some memory, and I/O (input/output) lines, all on one chip. The proper meaning of
microcomputer is a computer using a (number of) microprocessor(s) as its CPU(s), while the concept of
the patent is somewhat more similar to a micro controller.
11) Which of the following descriptions would NOT fit a microprocessor?
A) Electronic computer
B) Central Processing Unit
C) Memory disk
D) A single integrated chip circuit.
12) Select the TRUE statement from the following.
A) Microprocessors and computers on a chip are variations of each other.
B) Integration of processing power on chips has made processing power cheaper.
C) Before microprocessors, CPUs were not made from individual small scale ICs.
D) A microprocessor circuit only has transistors in it.
13) Which of the following was NOT the first to develop a microprocessor?
A) Microsoft
B) Intel
C) Texas Instruments
D) Garret
14) According to the passage, which of these is NOT a use of microprocessors?
A) Drivers for calculators
B) Core for large mainframes
C) Advanced mobile phones
D) Used for small handheld computers
15) "A number of microprocessors at its CPU" is an apt description of a:
A) 11icro-controller
B) Micro-computer
C) Micro-processor
D) Micro-transistor
Directions for Questions 16-20: Read the passage and answer the questions that follow on the basis of
the information provided in the passage.
Dynamic Link Libraries Windows provides several files called dynamic link libraries (DLLs) that
contain collections of software code that perform common functions such as opening or saving a file.
When Windows application wants to use one of those functions or routines, the app sends a message to
Windows with the names of the DLL file and the function. This procedure is known as calling a
function. One of the most frequently used DLLs is Windows COMMDLG.DLL, which includes among
others, the functions to display File Open, File Save, Search, and Print dialog boxes. The application
also sends any information the DLL function will need to complete the operation. For example, a
program calling the Open File function in COMMDLG.DLL would pass along a file spec, such as *. *
or *.DOC, to be displayed in the dialog box's Filename text box.
The application also passes along a specification for the type of information it expects the DLL to
return to the application when the DLL's work is done. The application, for example, may expect return
information in the form of integers, true/false values, or text. Windows passes the responsibility for
program execution to the DLL, along with the parameters and the return information the DLL will
need. The specific DLL is loaded into memory, and then executed by the processor. At this point the
DLL, rather than the application, runs things. The DLL performs all the operations necessary to
communicate with Windows and, through Windows, with the PC's hardware. After the DLL function is
complete, the DLL puts the return information into memory, where it can be found by the application,
and instructs Windows to remove the DLL routine from memory. The application inspects the return
information, which usually tells whether the DLL function was able to execute correctly. If the
operation was a success, the application continues from where it left off before issuing the function
call. If the operation failed, the application displays an error message.
16) By using DLLs, Windows:
A) Saves processing time
B) Multitasks
C) Shares program code
D) Communicates with PCs hardware
17) To use any routine of a DLL, Windows:
A) Searches and copies it in the application code and executes it
B) Loads the DLL file and searches and executes the routine
C) Loads just the required routine in memory and executes it
D) Searches the location of the routine and instructs the application to execute it
18) Which information does an application need to passto Windows to use a DLL routine?
A) Just the name of the routine
B) Just the name of the DLL, which finds in turn the routine to be executed in return
C) Both the name of the routine as well as DLL and any parameters
D) Name of the DLL, routine, any parameters and type of information to be returned
19) According to the passage, while the DLL routine is executing, the calling application:
A) Waits for the routine to execute
B) Continues with other tasks
C) Helps the DLL routine perform by communicating with Windows and through Windows with the
PC's hardware
D) Passes all responsibility of program execution to the DLL and is removed from memory
20) The DLL function after execution returns:
A) The parameters and information into memory, where it can be inspected by the calling application
B) Information into memory, where it can be inspected by the calling application
C) To the calling application the information required by it so that it can inspect it
D) The information required into memory so that DLL can inspect whether the function operation was
a success
Section 2 -Analytical Ability
No. of Questions: 20
Duration in Minutes: 20
21) 70 students are required to paint a picture. 52 use green color and some children use red, 38
students use both the colors. How many students use red color?
A) 24
B) 42
C) 56
D)70
22) At an international conference, 100 delegates spoke English, 40 spoke French, and 20 spoke both
English and French. How many delegates could speak at least one of these two languages?
A) 110
B) 100
C) 140
D) 120
23) A group of 50 students were required to clear 2 tasks, one in rock-climbing and the other in bridge
crossing during an adventure sports expedition. 30 students cleared both the tasks. 37 cleared bridge
crossing, 38 students cleared rockclimbing. How many students could not clear any task?
A)0
B)3
C)5
D) 9
24) A dance instructor conducts annual workshops in which he holds sessions for basic learners and
trainers. In a particular year, 2000 people attended the workshop. 1500 participated as learners and 800
as trainers. How many participated as only trainers?
A) 200
B) 500
C) 800
D) 1500
25) In a group of 400 readers who read science fiction or literacy works or both, 250 read science
fiction and 230 read literacy works. How many read both science fiction and literacy works?
A) 80
B) 160
C) 220
D) 400
26) A man said to a lady, "Your mother's husband's sister is my aunt". How is the lady related to the
man?
A) Daughter
B) Grand daughter
C) Mother
D) Sister
27) A man is facing west. He turns 45degree in the clockwise direction and then another 180 degree in
the same direction and then 270 degree in the anticlockwise direction. Which direction is he facing
now?
A) South
B) North-West
C) West
D) South-West
28) In a row of 60, if Ram is standing at 17th from the first, what is his position from the last?
A) 25
B) 43
C)44
D) 45
29) A man is facing northwest. He turns 90 degrees in the clockwise direction and then 135degrees in
the anti-clockwise direction. Which direction is he facing now?
A) East
B) West
C) North
D) South
30) What three letter word best completes the below words? VA - __E S___TER - - _ER
A) STR
B)TER
C) CAT
D) \\FAT
Directions for Questions 31-35: In the following questions mark: 1, if the question can be answered
with the help of statement I alone. 2, if the question can be answered with the help of statement II
alone. 3, if the question can be answered with the help of both I and II. 4, if the question cannot be
answered at all.
31) What is the value of P? I. P and Q are integers II. PQ = 10, P + Q =5
A)l
B)2
C)3
D)4
32) Who got the highest score in the Mathematics examination, among Sumit, Amit and Namit. No two
students got the same marks. I. Sumit got more marks than Namit. II. Amit did not get lesser marks
than Sumit, who did not get lesser marks than Namit.
A)l
B)2
C)3
D)4
33) How many hours does it take some boys and girls in a camp to put up the tent? I. There are 4 boys
and 7 girls. II. A girl can put up the tent in 5 hours and a boy can put up the tent in 3 hours.
A)l
B)2
C)3
D)4
34) If p, q, r, s and t are in an Arithmetic Progression, is r the largest among them? I.t>O II. p, q < 0
A)l
B)2
C)3
D)4
35) Is X a whole number, if X > O? I. 2X is an even number. II. 3X is an odd number.
A)l
B)2
C)3
D)4
Directions for Questions 36-40: In the following questions mark: 1, if the question can be answered
with the help of statement I alone. 2, if the question can be answered with the help of statement II
alone. 3, if the question can be answered with the help of both I and II. 4, if the question cannot be
answered at all.
31) What is the value of P? I. P and Q are integers II. PQ = 10, P + Q =5
A)l
B)2
C)3
D)4
32) Who got the highest score in the Mathematics examination, among Sumit, Amit and Namit. No two
students got the same marks. I. Sumit got more marks than Namit. II. Amit did not get lesser marks
than Sumit, who did not get lesser marks than Namit.
A)l
B)2
C)3
D)4
33) How many hours does it take some boys and girls in a camp to put up the tent? I. There are 4 boys
and 7 girls. II. A girl can put up the tent in 5 hours and a boy can put up the tent in 3 hours.
A)l
B)2
C)3
D)4
34) If p, q, r, s and t are in an Arithmetic Progression, is r the largest among them? I.t>O II. p, q < 0
A)l
B)2
C)3
D)4
35) Is X a whole number, if X > O? I. 2X is an even number. II. 3X is an odd number.
A)l
B)2
C)3
D)4
Directions for Questions 36-40: In a certain code, the symbol for 0 (zero) is. * and that for 1 is $. The
numb.:rs greater than 1 are to be written only by using the two symbols given above. The value of the
symbol for 1 doubles itself every time it shifts one place to the left. (For example, 4 is written as $**;
and; 3 is written as $$)
36) 11x 17 / 10 + 2 x 5 + 3 / 10 can also be represented as:
A) $*$$*
B) $*$$$
C) $$$*$
D) $**$$
37) 260 can be represented as:
A) $****$**
B) $$*$$$$$
C) $$*$$$$**
D) $*****$**
38) 60 / 17 can also be represented as:
A) $$$*$*** / $$**$$
B) $$$***** / $$**$$
C) $*$$*$** / $$**$$
D) $$*$*$** / $$**$$
39) $***$ can be represented as:
A) $$$ / $* B) $*$**- $$
C) $*$*$- $$
D) $$$***$ - $$
40) 30^2 can be represented as:
A) ($$*$$ ) $*+ $*$*$$*$
B) ($$*$$ ) $* + $$****$
C) ( $$*$$ ) $$ + $*$****
D) ( $$*$$ ) $$ + $*$**$
Section 3 : Attention To Detail
No. of Questions: 11
Duration in Minutes: 11
Directions for Questions 41-48: Follow the directions given below to answer the questions that follow.
Your answer for each question below would be: A, if ALL THREE items given in the question are
exactly ALIKE. B, if only the FIRST and SECOND items are exactly ALIKE. C, if only the FIRST and
THIRD items are exactly ALIKE. D, if only the SECOND and THIRD items are exactly ALIKE. E, if
ALL THREE items are DIFFERENT.
41)LLMLLLKLMPUU, LLMLLLKLMPUU, LLMLLLKLMPUU
A) A
B)B
C)C
D)D
E)E
42) 0452-9858762, 0452-9858762, 0452-9858762
A) A
B) B
C) C
D) D
E) E
43) NIINIININN, NIININNINN ,NIINIININN
A) A
B)B
C)C
D)D
E)E
44) 4665.8009291, 4665.7999291, 4665.8009291
A) A
B) B
C)C
D)D
E)E
45)808088080.8080, 808008080.8080, 808088080.8080
A) A
B)B
C)C
D)D
E)E
46) If* standsfor /, / stands for -,+ stands for * and -stands for +, then 9/8*7+5-10=?
A) 13.3
B) 10.8
C) 10.7
D) 11.4
47) If* stands for /, / stands for -,+ stands for * and -stands for +, then 9/15*9+2-9=?
A) 14.7
B) 15.3
C) 14.1
D) 16.2
48) If * stands for /, / stands for -, + stands for * and - stands for +, then which of the following is
TRUE?
A) 36/12*4+50-8 =-106
B) 12*8/4+50-8 =45.5
C) 36*4/12+36-8 = 4.7
D) 8*36/4+50-8 = 300
Set: 3648(A) ver-Z.O For: Aecenture IDC
Directions for Questions 49-51: In the following questions, the following letters indicate mathematical
operations as indicated below: A: Addition V: Equal to S: Subtraction W: Greater than M:
Multiplication X: Less than D: Division Out of the four alternatives given in these questions, only one
is coccect according to the above letter symbols. Identify the coccect one.
49) See the options given below
A) 6 S 7 A 2 M 3 W 0 D 7
B) 6 A 7 S 2 M 3 W 0 A 7
C) 6 S 7 M 2 S 3 W 0 M 7
D) 6 M 7 S 2 A 3 X 0 D 7
50) If * stands for -,/ stands for +, + stands for / and -stands for *, then which of the following is
TRUE?
A) 16/8*6+90-12 =23.2
B) 8*12/6+90-12 =7.2
C) 16*6/8+16-12 =-4.1
D) 12*16/6+90-12 =8
51) If * stands for -,/ stands for +, + stands for / and -stands for * , then which of the following is
TRUE?
A) 16*4/18+16-8 = -10.1
B) 18*8/4+40-8 =-2.8
C) 16/18*4+40-8 =33.2
D) 8*16/4+40-8 =-2
Directions for Questions 52-55: For the post of a manager of a leading call centre -Arkade Inc. -
situated in Ludhiana, the following are the criteria the candidate must satisfy: - The candidate should
have a Management Degree. - The candidate should have at least 4 years of similar experience atanother
call center. - The candidate should be more than 30 years of age as on the 1st of July 2003. -
The candidate should have 6 months of international exposure, i.e. should have been posted in a foreign
country. If a candidate does not satisfy the 1st condition but has more than 2 years of international
experience, then the VP operations, will interview him. If a candidate does not satisfy the 4th condition,
then the HR manager will interview him.
52) Shakuntala was selected for a managerial position in an international call center after she passed
out from AIM Management Institute. After working for 3 years in the call center, she took a sabbatical.
She is 29 years of age as on the date of application. She will be:
A) Interviewed only by the HR
B) Interviewed only by the VP
C) Rejected
D) Data insufficient
53) Rajiv has been working as a Manager in Zephyr Inc. for 4 years now. He is an Engineering
graduate from a premier engineering institute. His certificate lists his date of birth as 17/12/1974. He
has worked in the hotel industry at the executive level. He is:
A) Give an aptitude test
B) Interviewed by the VP
C) Data insufficient
D) Not considered
54) Soma has 2 years of experience in Welsh Inc. and 2 years of experience in Franc Inc., both leading
call centers, as a manager. She has a management degree from a premier management organization.
She turned 30 this December (2002). She is a B.Com Graduate from St. Xavier's, Calcutta. If she
applies for the post, she will:
A) Be interviewed directly by the VP Operations
B) Not be considered
C) Be interviewed by the HR
D) Have to give an aptitude test
55) Salina has over 4 years of experience in Care Touch, a leading call center, as a manager. She
completed her MBA from Ranchi and worked in Singapore for UNO for 2 years before joining Care
Touch. She will be:
A) Recruited
B) Rejected
C) Interviewed by dIe VP Operations
D) Data insufficient


Question1
If –2 (A) 1 (B) -14 (C) -7 (D) 1 (E) -24
Question 2
a, b, and c are integers and a < b < c. S is the set of all integers from a to b, inclusive. Q is the
set of all integers from b to c, inclusive. The median of set S is (3/4)b. The median of set Q is
(7/8)c. If R is the set of all integers from a to c, inclusive, what fraction of c is the median of set
R?
(A) 3/8
(B) 1/2
(C) 11/16
(D) 5/7
(E) ¾

Question 3
a, b, and c are positive integers. If a, b, and c are assembled into the six-digit number abcabc,
which one of the following must be a factor of abcabc?
(A) 16
(B) 13
(C) 5
(D) 3
(E) none of the above

Question 4
The average of (54,820)2 and (54,822)2 =
(A) (54,821)2
(B) (54,821.5)2
(C) (54,820.5)2
(D) (54,821)2 + 1
(E) (54,821)2 – 1

Question 5
A certain club has exactly 5 new members at the end of its first week. Every subsequent
week, each of the previous week's new members (and only these members) brings exactly x new
members into the club. If y is the number of new members brought into the club during the
twelfth week, which of the following could be y?
(A)
(B)
(C)
(D)
(E)

Question 6:
DATA SUFFICIENCY PROBLEM
Directions: The data sufficiency problem consists of a question and two statements, labeled (1)
and (2), in which certain data are given. You have to decide whether the data given in the
statements are sufficient for answering the question. Using the data given in the statements plus
your knowledge of mathematics and everyday facts (such as the number of days in July or the
meaning of counterclockwise), you must indicate whether
Choice: A statement (1) ALONE is sufficient, but statement (2) alone is not sufficient to
answer the question asked;
Choice: B statement (2) ALONE is sufficient, but statement (1) alone is not sufficient to
answer the question asked;
Choice: C BOTH statements (1) and (2) TO GETHER are sufficient to answer the question
asked, but NEITHER statement ALONE is sufficient;
Choice: D EACH statement ALONE is suffi cient to answer the question asked;
Choice: E statements (1} and (2) TOGETHER are NOT sufficient to answer the question
asked, and additional data specific to the problem are needed.
x is a positive number. If 9x + 9x+1 + 9x+2 + 9x+3 + 9x+4 + 9x+5 = y, is y divisible by 5?
1) 5 is a factor of x.
2) x is an integer.

Question 7
If the range of the set of numbers {150, 90, 125, 110, 170, 155, x, 100, 140} is 95, which of the
following could be x?
(A) 80
(B) 85
(C) 95
(D) 125
(E) 185

Question 8
Frances can complete a job in 12 hours, and Joan can complete the same job in 8 hours.
Frances starts the job at 9 a.m., and stops working at 3 p.m. If Joan starts working at 4 p.m. to
complete the job, at what time is the job finished?
(A) 6 p.m.
(B) 7 p.m.
(C) 8 p.m.
(D) 10 p.m.
(E) 12 p.m.

DATA SUFFICIENCY PROBLEM
Directions: The data sufficiency problem consists of a question and two statements, labeled (1)
and (2), in which certain data are given. You have to decide whether the data given in the
statements are sufficient for answering the question. Using the data given in the statements plus
your knowledge of mathematics and everyday facts (such as the number of days in July or the
meaning of counterclockwise), you must indicate whether
Choice: A statement (1) ALONE is sufficient, but statement (2) alone is not sufficient to
answer the question asked;
Choice: B statement (2) ALONE is sufficient, but statement (1) alone is not sufficient to
answer the question asked;
Choice: C BOTH statements (1) and (2) TO GETHER are sufficient to answer the question
asked, but NEITHER statement ALONE is sufficient;
Choice: D EACH statement ALONE is suffi cient to answer the question asked;
Choice: E statements (1} and (2) TOGETHER are NOT sufficient to answer the question
asked, and additional data specific to the problem are needed.

Question 9
A, B, C, D, and E are airline pilots with very busy travel schedules. Given that D is able to meet at
any time that B cannot meet, do the schedules of A, B, C, D, and E allow three of these five
individuals to meet together for two uninterrupted hours?
(1) Pilots A and C, who cannot meet together, are not able to end any meeting during the AM
hours of any weekday.
(2) Pilots B and E, who can never meet for longer than 2 uninterrupted hours, are only available
to meet for two straight hours starting at 10:30 PM on any weekday and not ending during the AM
hours of any weekend day.

Question 10.
1/212 + 2/213 + 3/214 + 4/215 =
(A) 1/210
(B) 1/212
(C) 15/215
(D) 2/210
(E) 23/216

Question 11
11+22+33+...+1010 is divided by 5. What is the remainder?
(A) 0
(B) 1
(C) 2
(D) 3
(E) 4

Question 12
If n is an integer greater than 0, what is the remainder when 912n+3 is divided by 10?
(A) 0
(B) 1
(C) 2
(D) 7
(E) 9

Question13
Each of 435 bags contains at least one of the following three items: raisins, almonds, and
peanuts. The number of bags that contain only raisins is 10 times the number of bags that contain
only peanuts. The number of bags that contain only almonds is 20 times the number of bags that
contain only raisins and peanuts. The number of bags that contain only peanuts is one-fifth the
number of bags that contain only almonds. 210 bags contain almonds. How many bags contain
only one kind of item?
(A) 256
(B) 260
(C) 316
(D) 320
(E) It cannot be determined from the given information.

Question 14
What is the probability that and are reciprocal fractions?
(1) v, w, y, and z are each randomly chosen from the first 100 positive integers.
(2) The product (u)(x) is the median of 100 consecutive integers.

Question 15
If n is an integer greater than 5.3, then n! must be divisible by which of the following numbers?
(A) 7
(B) 11
(C) 12
(D) 13
(E) 14

Question 16
what is population of the village
1. 7/11 of the village comprises of married people
2. 200 widows comprises 10% of the singles population
(A)
(B)
(C)
(D)
(E)

Question 17
The sum of the ages of A, B and C is 137 years. What is C’s age
1. the sum of the ages of A and C is 91 years
2. the sum of the ages of B and C is 104 years
(A)
(B)
(C)
(D)
(E)

Question 18
Is the number x is divisible by 9.
1. The number x can be obtained by taking the difference between a number and
another one obtained by reversing the digits.
2. The sum of the digits of x is divisible by 9.
(A)
(B)
(C)
(D)
(E)

Question 19
In an election how many votes are cast.
1. the winning candidate gets 54% of the total votes
2. the winning candidate wins by 800
(A)
(B)
(C)
(D)
(E)

Question 23.
Is x x b a 
A. 0 , x b a
B. x x
b a
,
1 1
(A)
(B)
(C)
(D)
(E)

Question 24
A printer numbered consecutively the pages of a book, beginning with 1 on the first page. In
numbering the pages, he had to print a total of 187 digits. Find the number of pages in
the book.
A) 99 B) 98 C) 96 D) 97 E) 95

Question 25
In a drawer of shirts, 8 r blue, 6 r green, and 4 r magenta. If Mason draws 2 shirts at random,
what is the probability that at least one of the shirts he draws will be blue?
(A) 25/153
(B) 28/153
(C) 5/17
(D) 4/9
(E) 12/17

Question 26
x2
+2x-8 =
x 2-6x+8
(A) 1
(B) -1
(C) x/3
(D) x+4
x-4
(E) x+8
x-8

Question 27
Which of the following CANNOT yield an integer when divided by 10?
(i) Product of two prime numbers
(ii) An integer less than 10
(iii) Sum of three consecutive integers
(iv) An odd integer
A. (i) & (iv) only
B. (ii) & (iv) only
C. (ii), (iii) & (iv) only
D. (iv) Only

Question 28
A number, K, is a positive integer with the special property that 3 times its unit is equal to 2 times
its 10 digit. How many such numbers exist between 10 & 99?
(A) 2
(B) 3
(C) 4
(D) 5

Question 29
If two digit integer M and N are positive and have same digits, but in reverse order, which of the
following cannot be the sum of M and N.
a) 181
b) 165
c) 121
d) 99
e) 44

Question 30
....
In the figure above, does x = 90?
(1) The length of AC is less than the length of BC.
(2) The length of AB is one-fourth the circumference of the circle.
(A)
(B)
(C)
(D)
(E)

Question: 31
In a group of 8 semifinalists, all but 2 will advance to the final round. If in the final round only the
top 3 will be awarded medals, then how many groups of medal winners are possible?
(A) 20
(B) 56
(C) 120
(D) 560
(E) 720

Question 32
The marks scored by a student in three subjects are in the ratio of 4: 5: 6. If the candidate scored
an overall aggregate of 60% of the sum of the maximum marks and the maximum mark in all
three subjects is the same, in how many subjects did he score more than 60%?
(A) 1 (B) 2
(C) 3 (D) None of the subjects

Question 33
There are 6 boxes numbered 1, 2,...6. Each box is to be filled up either with a red or a green ball
in such a way that at least 1 box contains a green ball and the boxes containing green balls are
consecutively numbered. The total number of ways in which this can be done is
(A) 5 (B) 21
(C) 33 (D) 60
(E) 40

Question 34
A vendor sells 60 percent of apples he had and throws away 15 percent of the remainder. Next
day he sells 50 percent of the remainder and throws away the rest. What percent of his apples
does the vendor throw?
(A) 17 (B) 23
(C) 77 (D) None of these
(E) 75

35. The product of two positive numbers is p. If each of the numbers is increased by 2, the new
product is how much greater than twice the sum of the two original numbers?
(A) p-2
(B) p
(C) p+2
(D) p+4
(E) 2p+4

36. If p and q are integers, such that p<0 I. 2p<2q
II. p2 III. p+q=0
(A) I only
(B) II only
(C) I and II only
(D) I and III only
(E) I, II and III

37. If 12(22)(35) is an integer, which of the following CANNOT be the value of p?
p
(A) 15
(B) 21
(C) 28
(D) 33
(E) 50

Solutions
Answer 1 (C). If we work with the two inequalities we subtract the max of 2nd from min
of 1st hence subtracting 11 from 3 gives us –8. Hence ineq. Should be –8 Answer 2 (C)
Since S contains only consecutive integers, its median is the average of the extreme
values a and b. We also know that the median of S is . We can set up and simplify
the following equation:
Since set Q contains only consecutive integers, its median is also the average of the
extreme values, in this case b and c. We also know that the median of Q is . We can
set up and simplify the following equation:
We can find the ratio of a to c as follows:
Taking the first equation,
and the second equation,
and setting them equal to each other, yields the following:
Since set R contains only consecutive integers, its median is the average of the extreme
values a and c: . We can use the ratio to substitute for a:
Thus the median of set R is . The correct answer is C.
Answer 3 (B)
This question looks daunting, but we can tackle it by thinking about the place values of
the unknowns. If we had a three-digit number abc, we could express it as 100a + 10b + c
(think of an example, say 375: 100(3) + 10(7) + 5). Thus, each additional digit increases
the place value tenfold.
If we have abcabc, we can express it as follows:
100000a + 10000b + 1000c + 100a + 10b + c
If we combine like terms, we get the following:
100100a + 10010b + 1001c
At this point, we can spot a pattern in the terms: each term is a multiple of 1001. On the
GMAT, such patterns are not accidental. If we factor 1001 from each term, the
expression can be simplified as follows:
1001(100a + 10b + c) or 1001(abc).
Thus, abcabc is the product of 1001 and abc, and will have all the factors of both. Since
we don't know the value of abc, we cannot know what its factors are. But we can see
whether one of the answer choices is a factor of 1001, which would make it a factor of
abcabc.
1001 is not even, so 16 is not a factor. 1001 doesn't end in 0 or 5, so 5 is not a factor.
The sum of the digits in 1001 is not a multiple of 3, so 3 is not a factor. It's difficult to
know whether 13 is a factor without performing the division: 1001/13 = 77. Since 13
divides into 1001 without a remainder, it is a factor of 1001 and thus a factor of abcabc.
The correct answer is B.
Answer 4 (D)
We can simplify this problem by using variables instead of numbers.
x = 54,820
x + 2 = 54,822
The average of (54,820)2 and (54,822)2 =
Now, factor x2 + 2x +2. This equals x2 + 2x +1 + 1, which equals (x + 1)2 + 1.
Substitute our original number back in for x as follows:
(x + 1)2 + 1 = (54,820 + 1)2 + 1 = (54,821)2 + 1.
The correct answer is D.
Answer 5 (D)
At the end of the first week, there are 5 members. During the second week, 5x new
members are brought in (x new members for every existing member). During the third
week, the previous week's new members (5x) each bring in x new members:
new members. If we continue this pattern to the twelfth week, we will see that new
members join the club that week. Since y is the number of new members joining during
week 12, .
If , we can set each of the answer choices equal to and see which one yields
an integer value (since y is a specific number of people, it must be an integer value). The
only choice to yield an integer value is (D):
Therefore x = 15.
Since choice (D) is the only one to yield an integer value, it is the correct answer.
Answer 6 (D)
The key to this problem is to recognize that in order for any integer to be divisible by 5, it
must end in 0 or 5. Since we are adding a string of powers of 9, the question becomes
"Does the sum of these powers of 9 end in 0 or 5?" If we knew the units digits of each
power of nine, we'd be able to figure out the units digit of their sum.
9 raised to an even exponent will result in a number whose units digit is 1 (e.g., 92 = 81,
94 = 6561, etc.). If 9 raised to an even exponent always gives 1 as the units digit, then 9
raised to an odd exponent will therefore result in a number whose units digit is 9 (think
about this: 92 = 81, so 93 will be 81 x 9 and the units digit will be 1 x 9).
Since our exponents in this case are x, x+1, x+2, x+3, x+4, and x+5, we need to know
whether x is an integer in order to be sure the pattern holds. (NEVER assume that an
unknown is an integer unless expressly informed). If x is in fact an integer, we will have
6 consecutive integers, of which 3 will necessarily be even and 3 odd. The 3 even
exponents will result in 1's and the 3 odd exponents will result in 9's. Since the three 1's
can be paired with the three 9's (for a sum of 30), the units digit of y will be 0 and y
will thus be divisible by 5. But we don't know whether x is an integer. For that, we need
to check the statements.
Statement (1) tells us that 5 is a factor of x, which means that x must be an integer.
Sufficient.
Statement (2) tells us that x is an integer. Sufficient.
The correct answer is D: EACH statement ALONE is sufficient to answer the question.
Answer 7 (E).
They tell us the range of the set {150, 90, 125, 110, 170, 155, x, 100, 140} is 95. Since
the present range without is 80, x has to be either the highest or the lowest number in the
set. If x is the lowest number, it would be 170-95=75, but that’s not an option. Therefore
x has to be the highest number. 90+95=185.
Answer 8 (C).
Plug in for number of tasks to be completed before the job is done; let the job involve
making 24 widgets. Thus, Frances makes 2 per hour, and Joan makes 3 per hour. Frances
works for 6 hours, so she makes 12 widgets. Joan needs 4 hours to make the other 12
widgets; if she starts at 4 p.m. she will finish at 8 p.m.
Answer 9 (E).
This is a Yes/No data sufficiency question. The only way the statements will provide
definitive information is if they lead to a definite YES answer or if they lead to a definite
NO answer. (A "Maybe" answer means that the statements are not sufficient.
Statement (1) alone only provides us information about when A and C cannot meet. It
does not provide any information about when each of the pilots ARE able to meet. While
we know that A and C cannot meet together, it is possible that some combination of three
pilots would be able to meet together (such as ABD or CBE). Statement (1) alone
therefore does not provide enough information to be able to definitively answer this
question YES or NO.
Statement (2) alone provides us with specific information about when B and E can meet.
However we are not provided with information as to whether one of the other pilots --A,
C or D -- will be able to join them for the meeting. Thus, statement (2) alone is not
sufficient to answer this question
In analyzing statements (1) and (2) together, it is helpful to list the 10 possible ways that
3 of the pilots could meet:
1. ABC
2. ABD
3. ABE
4. ACD
5. ACE
6. ADE
7. BCD
8. BCE
9. BDE
10. CDE
Statements (1) and (2) taken together preclude pilots A or C from meeting with pilots B
and E. This is due to the fact that pilots B and E can only meet for two straight hours
from 10:30 PM to 12:30 AM starting on either Monday, Tuesday, Wednesday, or
Thursday night while pilots A and C can never meet during the AM hours of any
weekday (leaving the 12:00 AM to 12:30 AM slot impossible). This eliminates 8 of the
10 possibilities (1, 2, 3, 5, 6 because A can't meet with B or E and 7, 8, 10 because C
can't meet with B or E.)
In addition, since pilot A cannot meet with pilot C, possibility 4 is also eliminated.
Thus, the only possibility that remains is #9: BDE. The question stem states that D is able
to meet at any time that B cannot. It may be tempting to use this information to conclude
that B and D are not able to meet together. However, while we know for sure that D is
able to meet at any time that B cannot, this does not preclude the possibility that D is
ALSO able to meet at times when B can meet. Given that we don't know whether or not
D can meet at the same time that B and E can meet - we do not have enough information
to evaluate whether pilots B, D, and E will be able to meet together.
Therefore, the correct answer is E: Statements (1) and (2) TOGETHER are NOT
sufficient.
Answer 10 (A).
If we express the numerators as powers of 2, then we would get 1/212 + 2/213 + 22/214 +
23/215 which is equal to 1/212 + 1/212 + 1/212 + 1/212 which equals 4/212. This can be
further reduced to 22/212 = ½10.
Answer 11(C)
When a whole number is divided by 5, the remainder depends on the units digit of that
number.
Thus, we need to determine the units digit of the number 11+22+33+...+1010. To do so, we
need to first determine the units digit of each of the individual terms in the expression as
follows:
Term Last (Units) Digit
11 1
22 4
33 7
44 6
55 5
66 6
77 3
88 6
99 9
1010 0
To determine the units digit of the expression itself, we must find the sum of all the units
digits of each of the individual terms:
1 + 4 + 7 + 6 + 5 + 6 + 3 + 6 + 9 = 47
Thus, 7 is the units digit of the number 11+22+33+...+1010. When an integer that ends in 7
is divided by 5, the remainder is 2. (Test this out on any integer ending in 7.)
Thus, the correct answer is C.
Answer 12 (E).
Look for the pattern: 91=9. 92=81. 93=729. Multiply that by another 9. You’ll get
another no ending in 1. And so forth and so on. So the bottom line is that whenever 9 is
raised to an odd power, the units digit is 9. When it’s raised to an even power, the units
digit is 1. When you divide a number by 10, its remainder will always be its units digit.
No matter what value you plug in for n, we’re always going to be raising 9 to an odd
power, so the units digit and the remainder will both be 9.
Answer 13 (D)
This problem involves 3 overlapping sets. To visualize a 3 set problem, it is best to draw
a Venn Diagram.
We can begin filling in our Venn Diagram utilizing the following 2 facts: (1) The number
of bags that contain only raisins is 10 times the number of bags that contain only peanuts.
(2) The number of bags that contain only almonds is 20 times the number of bags that
contain only raisins and peanuts.
Next, we are told that the number of bags that contain only peanuts (which we have
represented as x) is one- fifth the number of bags that contain only almonds (which we
have represented as 20y).
This yields the following equation: x = (1/5) 20y which simplifies to x = 4y. We can use
this information to revise our Venn Diagram by substituting any x in our original diagram
with 4y as in the figure.
Answer 14(C).
In order for one number to be the reciprocal of another number, their product must equal
1. Thus, this question can be rephrased as follows:
What is the probability that = 1?
This can be simplified as follows:
What is the probability that = 1 ?
What is the probability that = wz ?
Finally: What is the probability that ux = vywz ?
Statement (1) tells us that vywz is an integer, since it is the product of integers. However,
this gives no information about u and x and is therefore not sufficient to answer the
question.
Statement (2) tells us that ux is NOT an integer. This is because the median of an even
number of consecutive integers is NOT an integer. (For example, the median of 4
consecut ive integers - 9, 10, 11, 12 - equals 10.5.) However, this gives us no information
about vywz and is therefore not sufficient to answer the question.
Taking both statements together, we know that vywz IS an integer and that ux is NOT an
integer. Therefore vywz CANNOT be equal to ux. The probability that the fractions are
reciprocals is zero.
The correct answer is C: Statements (1) and (2) TAKEN TOGETHER are sufficient to
answer the question, but NEITHER statement ALONE is sufficient.
Answer 15 (C).
If n is greater than 5.3 then the smallest n! can be 6!. Since 6! =6*5*4*3*2*1, it is
definitely divisible by 12, because any n! bigger than 6 will include both a 6 and a 2, thus
making it a multiple of 12.Also, n! does not have to be divisible by anything greater than
6, so 7,11 and 13 are eliminated as are any multiples of those numbers, like 14.
Answer 16 (C).
The question here is to find out the population of the village. Statement (1) tells us that
7/11 of the village comprises of married people. So if the population of the village is x,
the no of married population is 7x/11. but this is absolutely not enough to get the total
population. So we have BCE. Now statement (2) tells that 200 widows comprise 10% of
the singles population. So the singles population is clearly 2000. so both the statements
(1) and (2) ALONE are not sufficient to answer the question. So now we combine both of
them. We get singles + widows + married = total population. Adding both statements we
get 2000+200+7x/11=x which gives us the total population of 6050 people. Therefore the
answer is (C).
Answer 17(C).
The question stem gives A+B+C=137. Now we look at the statement (1). It says A+C=91
which leads us to value of B=46. But it ALONE is not enough to get us the value of C.
Looking at statement (2) we get B+C=104 which leads us to value of A=33. This also
ALONE is not enough to get value of C. But both statements taken together will
definitely lead us to get the value of C from the question stem as 58. Therefore answer is
(C).
Answer 18(D).
We take the first statement and analyze it. Taking examples of 12 and 21 we get
difference of 9 that is divisible by 9. Again 13 and 31 gives difference of 18 divisible by
9. And so and so forth. So statement (1) ALONE is enough to answer the stem question.
So we have AD. Statement (2) gives us the divisibility rule of a number divisible by 9. So
it clearly answers the question asked in stem. So both statements ALONE are enough to
answer the question making (D) the right answer.
Answer 19(E).
The statement (1) here tells us that the winning candidate gets 54% of the total votes. But
this clearly is not enough to get the total vote count. So BDE. Statement (2) gives the
margin of victory but that too is not enough to answer the question involved. Even when
both the statements are taken together it leads us to nowhere. So some more data is
required to solve this problem clearly making (E) the right choice.
Answer 20(A).
If we expand the question stem we get ) 1 2 ( 2 x x
x
y
which can be reduced as xy +2y
+y/x. Now look at the statement (1). It tells that xy>0. If this is the case then surely the
whole equation becomes greater than 2y, which answers the question. So AD. Now
statement (2) gives us another equation that cannot help us solve the question asked in the
stem. So answer is (A).
Answer 21(E).
This is slightly tricky question. When square root of a number is involved the dividing
range becomes 0-1 or >1 since square root of a negative number is not included in
GMAT. So if a number is >1 then the question stem is true. And if the number is between
0-1 then the question stem is false. Statement (1) gives us the range as x>0 which takes
care of both our range. So nothing can be determined from this. So BCE. Statement (2)
tells us that x<2 which again covers both our range. So the answer is (E).
Answer 22(C).
To answer this question lets see the fact statements. Statement (1) says that x is +ve. This
ALONE is not enough to answer the stem question as x could be <1/5 and still be
positive. So BCE. Statement (2) can be transformed to the question stem but it could be
only true if x is positive. So clearly both the statements together will lead us to the answer
and hence (C).
Answer 23(E).
This question appears to be simple but it involves careful look at the question. The stem
question asks us for a relation. Statement (1) gives us the relation between a and b and
also the range of x. But the important point is that it has not given the range of a and b
whether they are positive or negative. Statement (2) also has the same problem as the
range for a and b are missing. So clearly answer is (E).
Answer 24(B).
The total number of digits is 187. The total number of single digit page numbers is
9(from 1 to 9). So subtracting this from 187 we get 178. After page number 9 we have 2
digit page numbers. So dividing this by 2 we get 89. So the total number of pages in the
book are 89+9=98.
Answer 25(E).
Remember that at least one is a clue, and when u see phrase, u need to find the
probability of getting everything except what u want (in other words, the probability of
getting any other color except blue), and then subtract that from 1. The formula for this
would be 1-(the probability of getting the other colors). 1-(10/18 * 9/17)=1-5/17=12/17.
Answer 26(D).
We don’t need to go about solving this question as per the REAL MATH way. We use
our technique. Whenever we see variables in the answer choice we just plug in!! Plug in
x=3, and the fraction becomes –7(target answer). Bingo!
Answer 27(D).
This question requires deep thinking in the sense that you have to look for the examples
to refute the statements. We will take one by one. If product of two prime numbers when
divided by 10 gives us an integer we can remove all answers containing (i). So we take 5
and 2. When multiplied and then divided by 10 we get an integer 1. So (i) is true. Get rid
of A. We move to (ii). Lets take 0. This when divided by 10 gives 0, which again is an
integer. So (ii) is also true. So get rid of B and C. Now we just need to verify D as it is the
obvious choice. Any odd integer when divided by 10 would always leave decimal and
never an integer. So D cannot be true. Hence it is the answer.
Answer 28(B).
Here’s another smart Question. It appears to be daunting but it’s not that tough. We start
with 1 at units place. When multiplied 3 times and then divided by 2 we get 1.5. So it is
ruled out. Next we try with 2. When we multiply by 3 we get 6 which when divided by 2
gives us 3.Bingo!! We get the first number 32. Similarly by trying out different numbers
at unit place we get other 2 numbers as 64 and 96(which are also multiples of 32 for
hint). So we get a total of 3 numbers between 10 and 99.
Answer 29(A).
Lets try this question by trial and error. Lets try to get all ans wers starting with smallest
value 44. It can be sum of 22 and 22. So (E) is ruled out. Now move to 99. It could be
sum of 54 and 45. So (D) goes. Next 121 could be written as 56 + 65. So even (C) goes.
Now try out 165. It could be the sum of 87 and 78. So after POE rest four (A) becomes
the answer, as it cannot be written as sum of desired combination.
Answer 30(B).
We need to see the fact statements. Statement (1) says that the length of AC is less than
the length of BC. This clearly leads us to nowhere. So BCE. Now the fact statement (2)
tells us that the length of AB is one- fourth the circumference of the circle, which clearly
leads us to know that it is not the diameter, it is just a chord. So the angle subtended is not
equal to 90 deg. So the answer is (B).
Answer 31 (B).
The entire discussion of rounds is a red herring. The question is asking for possible
combinations of the final 3, and it is possible for any of the original 8 contestants to have
advanced to the final round, thus we need to pick 3 out of 8, and order doesn’t matter.
8*7*6/3*2*1=56.
Answer 32(1).
Let the maximum in marks in each of the three subjects be 100.
Therefore, the candidate scored an aggregate of 60% of 3 * 100 = 60% of 300 marks =
180 marks.
Let the marks scored in the three subjects be 4x, 5x and 6x.
Then, 4x + 5x + 6x = 180
? 15x = 180 or x = 12.
Therefore, marks scored by the candidate in the three subjects are 4*12, 5*12 and 6*12
= 48, 60 and 72.
Hence, the candidate has score more than 60% in one subject.
Answer 33 (2).
If only one of the boxes has a green ball, it can be any of the 6 boxes. So, this can be
achieved in 6 ways.
If two of the boxes have green balls and then there are 5 consecutive sets of 2 boxes. 12,
23, 34, 45, 56.
Similarly, if 3 of the boxes have green balls, there will be 4 options.
If 4 boxes have green balls, there will be 3 options.
If 5 boxes have green balls, then there will be 2 options.
If all 6 boxes have green balls, then there will be just 1 option.
Total number of options = 6 + 5 + 4 + 3 + 2 + 1 = 21
Answer 34 (2).
Let the number of apples be 100.
On the first day he sells 60% apples ie.,60 apples.Remaining apples =40.
He throws 15% of the remaining i.e., 15% of 40 = 6.Now he has 40-6 = 34 apples
The next day he throws 50% of the remaining 34 apples i.e., 17.
Therefore in all he throws 6+17 =23 apples.
Answer 35(D)
Plug in p=10. This can be written as the product of 2 and 5. Their sum is 7. Twice of it is
14. Increasing the original numbers by 2 we get the two numbers as 4 and 7. The product
of this is 7*4=28. So the difference between the product and the sum is 28-14=14.This is
our target answer. Plug in p=10 back in all the answer choices and look for the answer as
14. (D) is the answer.
Answer 36(A)
Variables in the answer choices, plug in!! lets plug in p=-2 and q=2. Now verify all the
choices given. We get II false. So all the answers which contain II should go. So (B), (C)
and (E) go. Since two answer choices remain we plug in once more. Put p=-2 and q=3.
Now we can see that III is false. So the answer is (A).
Answer 37(E)
Factor out the numerator (4*3)(2*11)(5*7). Now this fraction when divided by p would
get an integer only if p cancels out with some of the factors. So now lets try it with
different answer choices. Except for the (E) all other choices can be cancelled out. So
clearly (E) is the answer.
__________________
Answered By StudyChaCha Member
Reply With Quote
  #3  
Old February 21st, 2015, 10:48 AM
Super Moderator
 
Join Date: Dec 2011
Default Re: Graduate Management Admission Test previous year question papers

Here I am providing the list of few questions of Graduate Management Admission Test exam paper which you are looking for .

Graduate Management Admission Test exam paper

Graduate Management Admission Test (GMAT)
Quantitative Section
? In the math section, you will have 75 minutes to answer 37 questions:
? A of these question are experimental and would not be counted toward your score.
There is no way to identify which questions are experimental as they one mixed in
randomly through out the section.
? This section includes two types of questions: Problem solving and Data sufficiency.
? For each problem solving section, you are to solve the problem and indicate the best of
the answer choices given.
? For each data sufficiency question, you are to decide whether the information given in
each of two statements, labeled (1) and (2), is sufficient to answer the question, whether
the information in the two statements together is sufficient, or if neither is sufficient.
? More detailed dissection will appear before the first occurrence of each question type. At
any point in the test, you can read the direction for the question type you working on by
clicking on HELP.
Quantitative Section
Time: 75 minutes
Solve the problem and indicate the best answer choices given.
Number: All the numbers used are Real Numbers
Figure: A figure accompanying used a problem solving is intended to provide information
useful in solving the problem. Figures are drawn as accurately as possible EXCEPT when it
is started in a specific problem that its figure is not drawn to scale.
Straight lines may some times appear jagged. All figure lie in a plane unless otherwise
indicated.
Question1
If –2 (A) 1 (B) -14 (C) -7 (D) 1 (E) -24 Question 2
a, b, and c are integers and a < b < c. S is the set of all integers from a to b, inclusive. Q is the
set of all integers from b to c, inclusive. The median of set S is (3/4)b. The median of set Q is
(7/8)c. If R is the set of all integers from a to c, inclusive, what fraction of c is the median of set
R?
(A) 3/8
(B) 1/2
(C) 11/16
(D) 5/7
(E) 3/4
Question 3
a, b, and c are positive integers. If a, b, and c are assembled into the six-digit number abcabc,
which one of the following must be a factor of abcabc?
(A) 16
(B) 13
(C) 5
(D) 3
(E) none of the above
Question 4
The average of (54,820)2 and (54,822)2 =
(A) (54,821)2
(B) (54,821.5)2
(C) (54,820.5)2
(D) (54,821)2 + 1
(E) (54,821)2 – 1
Question 5
A certain club has exactly 5 new members at the end of its first week. Every subsequent
week, each of the previous week's new members (and only these members) brings exactly x new
members into the club. If y is the number of new members brought into the club during the
twelfth week, which of the following could be y?
(A)
(B)
(C)
(D)
(E)
Question 6:
DATA SUFFICIENCY PROBLEM
Directions: The data sufficiency problem consists of a question and two statements, labeled (1)
and (2), in which certain data are given. You have to decide whether the data given in the
statements are sufficient for answering the question. Using the data given in the statements plus
your knowledge of mathematics and everyday facts (such as the number of days in July or the
meaning of counterclockwise), you must indicate whether
Choice: A statement (1) ALONE is sufficient, but statement (2) alone is not sufficient to
answer the question asked;
Choice: B statement (2) ALONE is sufficient, but statement (1) alone is not sufficient to
answer the question asked;
Choice: C BOTH statements (1) and (2) TO GETHER are sufficient to answer the question
asked, but NEITHER statement ALONE is sufficient;
Choice: D EACH statement ALONE is suffi cient to answer the question asked;
Choice: E statements (1} and (2) TOGETHER are NOT sufficient to answer the question
asked, and additional data specific to the problem are needed.
x is a positive number. If 9x + 9x+1 + 9x+2 + 9x+3 + 9x+4 + 9x+5 = y, is y divisible by 5?
1) 5 is a factor of x.
2) x is an integer.
Question 7
If the range of the set of numbers {150, 90, 125, 110, 170, 155, x, 100, 140} is 95, which of the
following could be x?
(A) 80
(B) 85
(C) 95
(D) 125
(E) 185
Question 8
Frances can complete a job in 12 hours, and Joan can complete the same job in 8 hours.
Frances starts the job at 9 a.m., and stops working at 3 p.m. If Joan starts working at 4 p.m. to
complete the job, at what time is the job finished?
(A) 6 p.m.
(B) 7 p.m.
(C) 8 p.m.
(D) 10 p.m.
(E) 12 p.m.
DATA SUFFICIENCY PROBLEM
Directions: The data sufficiency problem consists of a question and two statements, labeled (1)
and (2), in which certain data are given. You have to decide whether the data given in the
statements are sufficient for answering the question. Using the data given in the statements plus
your knowledge of mathematics and everyday facts (such as the number of days in July or the
meaning of counterclockwise), you must indicate whether
Choice: A statement (1) ALONE is sufficient, but statement (2) alone is not sufficient to
answer the question asked;
Choice: B statement (2) ALONE is sufficient, but statement (1) alone is not sufficient to
answer the question asked;
Choice: C BOTH statements (1) and (2) TO GETHER are sufficient to answer the question
asked, but NEITHER statement ALONE is sufficient;
Choice: D EACH statement ALONE is suffi cient to answer the question asked;
Choice: E statements (1} and (2) TOGETHER are NOT sufficient to answer the question
asked, and additional data specific to the problem are needed.
Question 9
A, B, C, D, and E are airline pilots with very busy travel schedules. Given that D is able to meet at
any time that B cannot meet, do the schedules of A, B, C, D, and E allow three of these five
individuals to meet together for two uninterrupted hours?
(1) Pilots A and C, who cannot meet together, are not able to end any meeting during the AM
hours of any weekday.
(2) Pilots B and E, who can never meet for longer than 2 uninterrupted hours, are only available
to meet for two straight hours starting at 10:30 PM on any weekday and not ending during the AM
hours of any weekend day.
Question 10.
1/212 + 2/213 + 3/214 + 4/215 =
(A) 1/210
(B) 1/212
(C) 15/215
(D) 2/210
(E) 23/216
Question 11
11+22+33+...+1010 is divided by 5. What is the remainder?
(A) 0
(B) 1
(C) 2
(D) 3
(E) 4
Question 12
If n is an integer greater than 0, what is the remainder when 912n+3 is divided by 10?
(A) 0
(B) 1
(C) 2
(D) 7
(E) 9
Question13
Each of 435 bags contains at least one of the following three items: raisins, almonds, and
peanuts. The number of bags that contain only raisins is 10 times the number of bags that contain
only peanuts. The number of bags that contain only almonds is 20 times the number of bags that
contain only raisins and peanuts. The number of bags that contain only peanuts is one-fifth the
number of bags that contain only almonds. 210 bags contain almonds. How many bags contain
only one kind of item?
(A) 256
(B) 260
(C) 316
(D) 320
(E) It cannot be determined from the given information.
Question 14
What is the probability that and are reciprocal fractions?
(1) v, w, y, and z are each randomly chosen from the first 100 positive integers.
(2) The product (u)(x) is the median of 100 consecutive integers.
Question 15
If n is an integer greater than 5.3, then n! must be divisible by which of the following numbers?
(A) 7
(B) 11
(C) 12
(D) 13
(E) 14
Question 16
what is population of the village
1. 7/11 of the village comprises of married people
2. 200 widows comprises 10% of the singles population
(A)
(B)
(C)
(D)
(E)
Question 17
The sum of the ages of A, B and C is 137 years. What is C’s age
1. the sum of the ages of A and C is 91 years
2. the sum of the ages of B and C is 104 years
(A)
(B)
(C)
(D)
(E)
Question 18
Is the number x is divisible by 9.
1. The number x can be obtained by taking the difference between a number and
another one obtained by reversing the digits.
2. The sum of the digits of x is divisible by 9.
(A)
(B)
(C)
(D)
(E)
Question 19
In an election how many votes are cast.
1. the winning candidate gets 54% of the total votes
2. the winning candidate wins by 800
(A)
(B)
(C)
(D)
(E)
Question 20
Is y x
x
y
2 ) 1 ( 2 ? ?
1. 0 ? xy
2. 5 ? ? y x
(A)
(B)
(C)
(D)
(E)
Question 21
Is
x
x
1
?
1. x >0
2. x < 2
(A)
(B)
(C)
(D)
(E)
Question 22
Is
5
1
? x
1. x is positive
2. 5
1
?
x
(A)
(B)
(C)
(D)
(E)
Question 23.
Is x x b a ?
A. 0 , ? ? x b a
B. x x
b a
? ? ,
1 1
(A)
(B)
(C)
(D)
(E)
Question 24
A printer numbered consecutively the pages of a book, beginning with 1 on the first page. In
numbering the pages, he had to print a total of 187 digits. Find the number of pages in
the book.
A) 99 B) 98 C) 96 D) 97 E) 95
Question 25
In a drawer of shirts, 8 r blue, 6 r green, and 4 r magenta. If Mason draws 2 shirts at random,
what is the probability that at least one of the shirts he draws will be blue?
(A) 25/153
(B) 28/153
(C) 5/17
(D) 4/9
(E) 12/17
Question 26
x2
+2x-8 =
x 2-6x+8
(A) 1
(B) -1
(C) x/3
(D) x+4
x-4
(E) x+8
x-8
Question 27
Which of the following CANNOT yield an integer when divided by 10?
(i) Product of two prime numbers
(ii) An integer less than 10
(iii) Sum of three consecutive integers
(iv) An odd integer
A. (i) & (iv) only
B. (ii) & (iv) only
C. (ii), (iii) & (iv) only
D. (iv) Only
Question 28
A number, K, is a positive integer with the special property that 3 times its unit is equal to 2 times
its 10 digit. How many such numbers exist between 10 & 99?
(A) 2
(B) 3
(C) 4
(D) 5
Question 29
If two digit integer M and N are positive and have same digits, but in reverse order, which of the
following cannot be the sum of M and N.
a) 181
b) 165
c) 121
d) 99
e) 44
Question 30
....
In the figure above, does x = 90?
(1) The length of AC is less than the length of BC.
(2) The length of AB is one-fourth the circumference of the circle.
(A)
(B)
(C)
(D)
(E)
Question: 31
In a group of 8 semifinalists, all but 2 will advance to the final round. If in the final round only the
top 3 will be awarded medals, then how many groups of medal winners are possible?
(A) 20
(B) 56
(C) 120
(D) 560
(E) 720
Question 32
The marks scored by a student in three subjects are in the ratio of 4: 5: 6. If the candidate scored
an overall aggregate of 60% of the sum of the maximum marks and the maximum mark in all
three subjects is the same, in how many subjects did he score more than 60%?
(A) 1 (B) 2
(C) 3 (D) None of the subjects
Question 33
There are 6 boxes numbered 1, 2,...6. Each box is to be filled up either with a red or a green ball
in such a way that at least 1 box contains a green ball and the boxes containing green balls are
consecutively numbered. The total number of ways in which this can be done is
(A) 5 (B) 21
(C) 33 (D) 60
(E) 40
Question 34
A vendor sells 60 percent of apples he had and throws away 15 percent of the remainder. Next
day he sells 50 percent of the remainder and throws away the rest. What percent of his apples
does the vendor throw?
(A) 17 (B) 23
(C) 77 (D) None of these
(E) 75
35. The product of two positive numbers is p. If each of the numbers is increased by 2, the new
product is how much greater than twice the sum of the two original numbers?
(A) p-2
(B) p
(C) p+2
(D) p+4
(E) 2p+4
36. If p and q are integers, such that p<0 I. 2p<2q
II. p2 III. p+q=0
(A) I only
(B) II only
(C) I and II only
(D) I and III only
(E) I, II and III
37. If 12(22)(35) is an integer, which of the following CANNOT be the value of p?
p
(A) 15
(B) 21
(C) 28
(D) 33
(E) 50
Solutions
Answer 1 (C). If we work with the two inequalities we subtract the max of 2nd from min
of 1st hence subtracting 11 from 3 gives us –8. Hence ineq. Should be –8 Answer 2 (C)
Since S contains only consecutive integers, its median is the average of the extreme
values a and b. We also know that the median of S is . We can set up and simplify
the following equation:
Since set Q contains only consecutive integers, its median is also the average of the
extreme values, in this case b and c. We also know that the median of Q is . We can
set up and simplify the following equation:
We can find the ratio of a to c as follows:
Taking the first equation,
and the second equation,
and setting them equal to each other, yields the following:
Since set R contains only consecutive integers, its median is the average of the extreme
values a and c: . We can use the ratio to substitute for a:
Thus the median of set R is . The correct answer is C.
Answer 3 (B)
This question looks daunting, but we can tackle it by thinking about the place values of
the unknowns. If we had a three-digit number abc, we could express it as 100a + 10b + c
(think of an example, say 375: 100(3) + 10(7) + 5). Thus, each additional digit increases
the place value tenfold.
If we have abcabc, we can express it as follows:
100000a + 10000b + 1000c + 100a + 10b + c
If we combine like terms, we get the following:
100100a + 10010b + 1001c
At this point, we can spot a pattern in the terms: each term is a multiple of 1001. On the
GMAT, such patterns are not accidental. If we factor 1001 from each term, the
expression can be simplified as follows:
1001(100a + 10b + c) or 1001(abc).
Thus, abcabc is the product of 1001 and abc, and will have all the factors of both. Since
we don't know the value of abc, we cannot know what its factors are. But we can see
whether one of the answer choices is a factor of 1001, which would make it a factor of
abcabc.
1001 is not even, so 16 is not a factor. 1001 doesn't end in 0 or 5, so 5 is not a factor.
The sum of the digits in 1001 is not a multiple of 3, so 3 is not a factor. It's difficult to
know whether 13 is a factor without performing the division: 1001/13 = 77. Since 13
divides into 1001 without a remainder, it is a factor of 1001 and thus a factor of abcabc.
The correct answer is B.
Answer 4 (D)
We can simplify this problem by using variables instead of numbers.
x = 54,820
x + 2 = 54,822
The average of (54,820)2 and (54,822)2 =
Now, factor x2 + 2x +2. This equals x2 + 2x +1 + 1, which equals (x + 1)2 + 1.
Substitute our original number back in for x as follows:
(x + 1)2 + 1 = (54,820 + 1)2 + 1 = (54,821)2 + 1.
The correct answer is D.
Answer 5 (D)
At the end of the first week, there are 5 members. During the second week, 5x new
members are brought in (x new members for every existing member). During the third
week, the previous week's new members (5x) each bring in x new members:
new members. If we continue this pattern to the twelfth week, we will see that new
members join the club that week. Since y is the number of new members joining during
week 12, .
If , we can set each of the answer choices equal to and see which one yields
an integer value (since y is a specific number of people, it must be an integer value). The
only choice to yield an integer value is (D):
Therefore x = 15.
Since choice (D) is the only one to yield an integer value, it is the correct answer.
Answer 6 (D)
The key to this problem is to recognize that in order for any integer to be divisible by 5, it
must end in 0 or 5. Since we are adding a string of powers of 9, the question becomes
"Does the sum of these powers of 9 end in 0 or 5?" If we knew the units digits of each
power of nine, we'd be able to figure out the units digit of their sum.
9 raised to an even exponent will result in a number whose units digit is 1 (e.g., 92 = 81,
94 = 6561, etc.). If 9 raised to an even exponent always gives 1 as the units digit, then 9
raised to an odd exponent will therefore result in a number whose units digit is 9 (think
about this: 92 = 81, so 93 will be 81 x 9 and the units digit will be 1 x 9).
Since our exponents in this case are x, x+1, x+2, x+3, x+4, and x+5, we need to know
whether x is an integer in order to be sure the pattern holds. (NEVER assume that an
unknown is an integer unless expressly informed). If x is in fact an integer, we will have
6 consecutive integers, of which 3 will necessarily be even and 3 odd. The 3 even
exponents will result in 1's and the 3 odd exponents will result in 9's. Since the three 1's
can be paired with the three 9's (for a sum of 30), the units digit of y will be 0 and y
will thus be divisible by 5. But we don't know whether x is an integer. For that, we need
to check the statements.
Statement (1) tells us that 5 is a factor of x, which means that x must be an integer.
Sufficient.
Statement (2) tells us that x is an integer. Sufficient.
The correct answer is D: EACH statement ALONE is sufficient to answer the question.
Answer 7 (E).
They tell us the range of the set {150, 90, 125, 110, 170, 155, x, 100, 140} is 95. Since
the present range without is 80, x has to be either the highest or the lowest number in the
set. If x is the lowest number, it would be 170-95=75, but that’s not an option. Therefore
x has to be the highest number. 90+95=185.
Answer 8 (C).
Plug in for number of tasks to be completed before the job is done; let the job involve
making 24 widgets. Thus, Frances makes 2 per hour, and Joan makes 3 per hour. Frances
works for 6 hours, so she makes 12 widgets. Joan needs 4 hours to make the other 12
widgets; if she starts at 4 p.m. she will finish at 8 p.m.
Answer 9 (E).
This is a Yes/No data sufficiency question. The only way the statements will provide
definitive information is if they lead to a definite YES answer or if they lead to a definite
NO answer. (A "Maybe" answer means that the statements are not sufficient.
Statement (1) alone only provides us information about when A and C cannot meet. It
does not provide any information about when each of the pilots ARE able to meet. While
we know that A and C cannot meet together, it is possible that some combination of three
pilots would be able to meet together (such as ABD or CBE). Statement (1) alone
therefore does not provide enough information to be able to definitively answer this
question YES or NO.
Statement (2) alone provides us with specific information about when B and E can meet.
However we are not provided with information as to whether one of the other pilots --A,
C or D -- will be able to join them for the meeting. Thus, statement (2) alone is not
sufficient to answer this question
In analyzing statements (1) and (2) together, it is helpful to list the 10 possible ways that
3 of the pilots could meet:
1. ABC
2. ABD
3. ABE
4. ACD
5. ACE
6. ADE
7. BCD
8. BCE
9. BDE
10. CDE
Statements (1) and (2) taken together preclude pilots A or C from meeting with pilots B
and E. This is due to the fact that pilots B and E can only meet for two straight hours
from 10:30 PM to 12:30 AM starting on either Monday, Tuesday, Wednesday, or
Thursday night while pilots A and C can never meet during the AM hours of any
weekday (leaving the 12:00 AM to 12:30 AM slot impossible). This eliminates 8 of the
10 possibilities (1, 2, 3, 5, 6 because A can't meet with B or E and 7, 8, 10 because C
can't meet with B or E.)
In addition, since pilot A cannot meet with pilot C, possibility 4 is also eliminated.
Thus, the only possibility that remains is #9: BDE. The question stem states that D is able
to meet at any time that B cannot. It may be tempting to use this information to conclude
that B and D are not able to meet together. However, while we know for sure that D is
able to meet at any time that B cannot, this does not preclude the possibility that D is
ALSO able to meet at times when B can meet. Given that we don't know whether or not
D can meet at the same time that B and E can meet - we do not have enough information
to evaluate whether pilots B, D, and E will be able to meet together.
Therefore, the correct answer is E: Statements (1) and (2) TOGETHER are NOT
sufficient.
Answer 10 (A).
If we express the numerators as powers of 2, then we would get 1/212 + 2/213 + 22/214 +
23/215 which is equal to 1/212 + 1/212 + 1/212 + 1/212 which equals 4/212. This can be
further reduced to 22/212 = ½10.
Answer 11(C)
When a whole number is divided by 5, the remainder depends on the units digit of that
number.
Thus, we need to determine the units digit of the number 11+22+33+...+1010. To do so, we
need to first determine the units digit of each of the individual terms in the expression as
follows:
Term Last (Units) Digit
11 1
22 4
33 7
44 6
55 5
66 6
77 3
88 6
99 9
1010 0
To determine the units digit of the expression itself, we must find the sum of all the units
digits of each of the individual terms:
1 + 4 + 7 + 6 + 5 + 6 + 3 + 6 + 9 = 47
Thus, 7 is the units digit of the number 11+22+33+...+1010. When an integer that ends in 7
is divided by 5, the remainder is 2. (Test this out on any integer ending in 7.)
Thus, the correct answer is C.
Answer 12 (E).
Look for the pattern: 91=9. 92=81. 93=729. Multiply that by another 9. You’ll get
another no ending in 1. And so forth and so on. So the bottom line is that whenever 9 is
raised to an odd power, the units digit is 9. When it’s raised to an even power, the units
digit is 1. When you divide a number by 10, its remainder will always be its units digit.
No matter what value you plug in for n, we’re always going to be raising 9 to an odd
power, so the units digit and the remainder will both be 9.
Answer 13 (D)
This problem involves 3 overlapping sets. To visualize a 3 set problem, it is best to draw
a Venn Diagram.
We can begin filling in our Venn Diagram utilizing the following 2 facts: (1) The number
of bags that contain only raisins is 10 times the number of bags that contain only peanuts.
(2) The number of bags that contain only almonds is 20 times the number of bags that
contain only raisins and peanuts.
Next, we are told that the number of bags that contain only peanuts (which we have
represented as x) is one- fifth the number of bags that contain only almonds (which we
have represented as 20y).
This yields the following equation: x = (1/5) 20y which simplifies to x = 4y. We can use
this information to revise our Venn Diagram by substituting any x in our original diagram
with 4y as in the figure.
Answer 14(C).
In order for one number to be the reciprocal of another number, their product must equal
1. Thus, this question can be rephrased as follows:
What is the probability that = 1?
This can be simplified as follows:
What is the probability that = 1 ?
What is the probability that = wz ?
Finally: What is the probability that ux = vywz ?
Statement (1) tells us that vywz is an integer, since it is the product of integers. However,
this gives no information about u and x and is therefore not sufficient to answer the
question.
Statement (2) tells us that ux is NOT an integer. This is because the median of an even
number of consecutive integers is NOT an integer. (For example, the median of 4
consecut ive integers - 9, 10, 11, 12 - equals 10.5.) However, this gives us no information
about vywz and is therefore not sufficient to answer the question.
Taking both statements together, we know that vywz IS an integer and that ux is NOT an
integer. Therefore vywz CANNOT be equal to ux. The probability that the fractions are
reciprocals is zero.
The correct answer is C: Statements (1) and (2) TAKEN TOGETHER are sufficient to
answer the question, but NEITHER statement ALONE is sufficient.
Answer 15 (C).
If n is greater than 5.3 then the smallest n! can be 6!. Since 6! =6*5*4*3*2*1, it is
definitely divisible by 12, because any n! bigger than 6 will include both a 6 and a 2, thus
making it a multiple of 12.Also, n! does not have to be divisible by anything greater than
6, so 7,11 and 13 are eliminated as are any multiples of those numbers, like 14.
Answer 16 (C).
The question here is to find out the population of the village. Statement (1) tells us that
7/11 of the village comprises of married people. So if the population of the village is x,
the no of married population is 7x/11. but this is absolutely not enough to get the total
population. So we have BCE. Now statement (2) tells that 200 widows comprise 10% of
the singles population. So the singles population is clearly 2000. so both the statements
(1) and (2) ALONE are not sufficient to answer the question. So now we combine both of
them. We get singles + widows + married = total population. Adding both statements we
get 2000+200+7x/11=x which gives us the total population of 6050 people. Therefore the
answer is (C).
Answer 17(C).
The question stem gives A+B+C=137. Now we look at the statement (1). It says A+C=91
which leads us to value of B=46. But it ALONE is not enough to get us the value of C.
Looking at statement (2) we get B+C=104 which leads us to value of A=33. This also
ALONE is not enough to get value of C. But both statements taken together will
definitely lead us to get the value of C from the question stem as 58. Therefore answer is
(C).
Answer 18(D).
We take the first statement and analyze it. Taking examples of 12 and 21 we get
difference of 9 that is divisible by 9. Again 13 and 31 gives difference of 18 divisible by
9. And so and so forth. So statement (1) ALONE is enough to answer the stem question.
So we have AD. Statement (2) gives us the divisibility rule of a number divisible by 9. So
it clearly answers the question asked in stem. So both statements ALONE are enough to
answer the question making (D) the right answer.
Answer 19(E).
The statement (1) here tells us that the winning candidate gets 54% of the total votes. But
this clearly is not enough to get the total vote count. So BDE. Statement (2) gives the
margin of victory but that too is not enough to answer the question involved. Even when
both the statements are taken together it leads us to nowhere. So some more data is
required to solve this problem clearly making (E) the right choice.
Answer 20(A).
If we expand the question stem we get ) 1 2 ( 2 ? ? x x
x
y
which can be reduced as xy +2y
+y/x. Now look at the statement (1). It tells that xy>0. If this is the case then surely the
whole equation becomes greater than 2y, which answers the question. So AD. Now
statement (2) gives us another equation that cannot help us solve the question asked in the
stem. So answer is (A).
Answer 21(E).
This is slightly tricky question. When square root of a number is involved the dividing
range becomes 0-1 or >1 since square root of a negative number is not included in
GMAT. So if a number is >1 then the question stem is true. And if the number is between
0-1 then the question stem is false. Statement (1) gives us the range as x>0 which takes
care of both our range. So nothing can be determined from this. So BCE. Statement (2)
tells us that x<2 which again covers both our range. So the answer is (E).
Answer 22(C).
To answer this question lets see the fact statements. Statement (1) says that x is +ve. This
ALONE is not enough to answer the stem question as x could be <1/5 and still be
positive. So BCE. Statement (2) can be transformed to the question stem but it could be
only true if x is positive. So clearly both the statements together will lead us to the answer
and hence (C).
Answer 23(E).
This question appears to be simple but it involves careful look at the question. The stem
question asks us for a relation. Statement (1) gives us the relation between a and b and
also the range of x. But the important point is that it has not given the range of a and b
whether they are positive or negative. Statement (2) also has the same problem as the
range for a and b are missing. So clearly answer is (E).
Answer 24(B).
The total number of digits is 187. The total number of single digit page numbers is
9(from 1 to 9). So subtracting this from 187 we get 178. After page number 9 we have 2
digit page numbers. So dividing this by 2 we get 89. So the total number of pages in the
book are 89+9=98.
Answer 25(E).
Remember that at least one is a clue, and when u see phrase, u need to find the
probability of getting everything except what u want (in other words, the probability of
getting any other color except blue), and then subtract that from 1. The formula for this
would be 1-(the probability of getting the other colors). 1-(10/18 * 9/17)=1-5/17=12/17.
Answer 26(D).
We don’t need to go about solving this question as per the REAL MATH way. We use
our technique. Whenever we see variables in the answer choice we just plug in!! Plug in
x=3, and the fraction becomes –7(target answer). Bingo!
Answer 27(D).
This question requires deep thinking in the sense that you have to look for the examples
to refute the statements. We will take one by one. If product of two prime numbers when
divided by 10 gives us an integer we can remove all answers containing (i). So we take 5
and 2. When multiplied and then divided by 10 we get an integer 1. So (i) is true. Get rid
of A. We move to (ii). Lets take 0. This when divided by 10 gives 0, which again is an
integer. So (ii) is also true. So get rid of B and C. Now we just need to verify D as it is the
obvious choice. Any odd integer when divided by 10 would always leave decimal and
never an integer. So D cannot be true. Hence it is the answer.
Answer 28(B).
Here’s another smart Question. It appears to be daunting but it’s not that tough. We start
with 1 at units place. When multiplied 3 times and then divided by 2 we get 1.5. So it is
ruled out. Next we try with 2. When we multiply by 3 we get 6 which when divided by 2
gives us 3.Bingo!! We get the first number 32. Similarly by trying out different numbers
at unit place we get other 2 numbers as 64 and 96(which are also multiples of 32 for
hint). So we get a total of 3 numbers between 10 and 99.
Answer 29(A).
Lets try this question by trial and error. Lets try to get all ans wers starting with smallest
value 44. It can be sum of 22 and 22. So (E) is ruled out. Now move to 99. It could be
sum of 54 and 45. So (D) goes. Next 121 could be written as 56 + 65. So even (C) goes.
Now try out 165. It could be the sum of 87 and 78. So after POE rest four (A) becomes
the answer, as it cannot be written as sum of desired combination.
Answer 30(B).
We need to see the fact statements. Statement (1) says that the length of AC is less than
the length of BC. This clearly leads us to nowhere. So BCE. Now the fact statement (2)
tells us that the length of AB is one- fourth the circumference of the circle, which clearly
leads us to know that it is not the diameter, it is just a chord. So the angle subtended is not
equal to 90 deg. So the answer is (B).
Answer 31 (B).
The entire discussion of rounds is a red herring. The question is asking for possible
combinations of the final 3, and it is possible for any of the original 8 contestants to have
advanced to the final round, thus we need to pick 3 out of 8, and order doesn’t matter.
8*7*6/3*2*1=56.
Answer 32(1).
Let the maximum in marks in each of the three subjects be 100.
Therefore, the candidate scored an aggregate of 60% of 3 * 100 = 60% of 300 marks =
180 marks.
Let the marks scored in the three subjects be 4x, 5x and 6x.
Then, 4x + 5x + 6x = 180
? 15x = 180 or x = 12.
Therefore, marks scored by the candidate in the three subjects are 4*12, 5*12 and 6*12
= 48, 60 and 72.
Hence, the candidate has score more than 60% in one subject.
Answer 33 (2).
If only one of the boxes has a green ball, it can be any of the 6 boxes. So, this can be
achieved in 6 ways.
If two of the boxes have green balls and then there are 5 consecutive sets of 2 boxes. 12,
23, 34, 45, 56.
Similarly, if 3 of the boxes have green balls, there will be 4 options.
If 4 boxes have green balls, there will be 3 options.
If 5 boxes have green balls, then there will be 2 options.
If all 6 boxes have green balls, then there will be just 1 option.
Total number of options = 6 + 5 + 4 + 3 + 2 + 1 = 21
Answer 34 (2).
Let the number of apples be 100.
On the first day he sells 60% apples ie.,60 apples.Remaining apples =40.
He throws 15% of the remaining i.e., 15% of 40 = 6.Now he has 40-6 = 34 apples
The next day he throws 50% of the remaining 34 apples i.e., 17.
Therefore in all he throws 6+17 =23 apples.
Answer 35(D)
Plug in p=10. This can be written as the product of 2 and 5. Their sum is 7. Twice of it is
14. Increasing the original numbers by 2 we get the two numbers as 4 and 7. The product
of this is 7*4=28. So the difference between the product and the sum is 28-14=14.This is
our target answer. Plug in p=10 back in all the answer choices and look for the answer as
14. (D) is the answer.
Answer 36(A)
Variables in the answer choices, plug in!! lets plug in p=-2 and q=2. Now verify all the
choices given. We get II false. So all the answers which contain II should go. So (B), (C)
and (E) go. Since two answer choices remain we plug in once more. Put p=-2 and q=3.
Now we can see that III is false. So the answer is (A).
Answer 37(E)
Factor out the numerator (4*3)(2*11)(5*7). Now this fraction when divided by p would
get an integer only if p cancels out with some of the factors. So now lets try it with
different answer choices. Except for the (E) all other choices can be cancelled out. So
clearly (E) is the answer.
__________________
Answered By StudyChaCha Member
Reply With Quote
Reply




All times are GMT +6. The time now is 09:43 AM.


Powered by vBulletin® Version 3.8.11
Copyright ©2000 - 2024, vBulletin Solutions Inc.
Search Engine Friendly URLs by vBSEO 3.6.0 PL2

1 2 3 4 5 6 7 8